Recent content by zero13428

  1. Z

    Lorentz Transformations and Reference Frames Problem

    Homework Statement In the old West, a marshal riding on a train traveling 35.0 m/s sees a duel between two men standing on the Earth 55.0 m apart parallel to the train. The marshal's instruments indicate that in his reference frame the two men fire simultaneously. (a) Which of the two men, the...
  2. Z

    Uncertainty in area of a circle

    Actually I think I got it worked out. Let me know if this looks right. A=(∏)(r)^2 ∂(A)/∂(r) = 2(∏)(r) sigma_A=√(((∂A/∂r)^2)(sigma_r)^2)) sigma_A=√(((2∏(14.3))^2)(0.3)^2))= 26.9cm Area = 642.4cm Uncertainty = 26.9cm
  3. Z

    Uncertainty in area of a circle

    I know at the beginning I asked how to use sigma and partial derivatives to solve this type of problem but I don't really know much about them yet. We haven't gotten to them in my math class. This problem is coming from an intro to physics lab course that focuses on propagation of error and...
  4. Z

    Uncertainty in area of a circle

    You said to take the ln of both sides. As in the natural log? I didn't know these had anything to logs or am I reading something wrong.
  5. Z

    Uncertainty in area of a circle

    Homework Statement The radius of a circle is measured to be 14.3+-0.3cm. Find the circle's area and the uncertainty in the area. I don't understand how to correctly apply uncertainty equations with sigma and partial derivatives to these types of problems. Homework Equations...
  6. Z

    Use comparison theorem to show if integral is convergent or divergent

    Ok, so if I can break the function into two parts, as long as one part (in this case 1/x) converges or diverges then the entire function does? I thought I could only do these type of integrals by picking a function that was larger or smaller and then comparing.
  7. Z

    Use comparison theorem to show if integral is convergent or divergent

    Homework Statement int (e^-x)/(x)dx from 0 to infinity Determine if integral is convergent or divergent2. The attempt at a solution I assume because the bottom limit is 0 and there is an x in the bottom of the integral that this is going to be divergent but I still have to use the...
Back
Top